Đến nội dung

Hình ảnh

[MARATHON] Chuyên đề Bất đẳng thức

bất đẳng thức marathon cấp 2 cấp 3 imo vmo olympic tuyển sinh 10

  • Please log in to reply
Chủ đề này có 85 trả lời

#1
pcoVietnam02

pcoVietnam02

    Thượng sĩ

  • Thành viên
  • 208 Bài viết

Xin chào, mình là pcoVienam02. Như các bạn có thể thấy thì hiện tại trên Diễn đàn đang có nhiều TOPIC, nhưng mà nó có thể làm các bạn học hơi khô khan. Nên mình sẽ cải biến TOPIC thành một loại mới, chính là Marathon.

Vậy Marathon là gì?

Marathon (mình sẽ lấy format từ diễn đàn mình đang làm việc - AoPS), gồm 2 thể loại chính:

Marathon loại 1 tức là người đăng chủ đề sẽ gửi bài toán đầu tiên (bài toán gốc). Người nào giải được bài toán gốc sẽ tiếp tục đưa ra câu hỏi thứ hai để những người giải được sau đó sẽ đưa ra câu hỏi tiếp theo, và cứ liên tục như thế.

Marathon loại 2 là người đăng chủ đề sẽ là người chấm điểm, và có nhiệm vụ gửi các bài toán theo thứ tự (mỗi lần 1 bài), ai giải được sẽ được 1 điểm (người giải sớm nhất). Nếu ai giải sai mà có người chỉ được điểm sai sót trước khi người đăng đáp án bài đó nhận ra sẽ được 0,5đ. Sau một số hữu hạn bài (thường là 100-200 bài) thì ai có số điểm cao hơn thì sẽ chiến thắng. 

Thì loại 1 chỉ mang tính chất học hỏi và cũng có khá nhiều rủi ro vì nếu có người gửi bài quá khó thì Marathon coi như chấm dứt. Vì vậy dựa trên tình hình diễn đàn thì mình sẽ tổ chức Marathon loại 2 cho các bạn vì mục đích vừa học hỏi vừa có sự thi đua giữa các bạn của 3 miền Tổ Quốc. 

 

*Lưu ý: Thời hạn giải mỗi bài là 2 ngày. 

Để khai mạc kì Marathon phiên bản mới mình sẽ 'khui' bài tập đầu tiên (khá dễ):

$\boxed{1}$ Cho $a,b,c$ là các số dương thỏa

$\frac{a}{1+a}+\frac{b}{1+b}+\frac{c}{1+c}=1$
Chứng minh rằng:  $abc \leq \frac{1}{8}$
 
============

Bảng điểm:* (Update lần 7)

Hoang72: 5 điểm

KietLW9: 2 điểm

yungazier: 1 điểm  

ChiMiwhh: 1 điểm

 

*Bảng điểm và người làm bài gần nhất sẽ được update sớm


Bài viết đã được chỉnh sửa nội dung bởi pcoVietnam02: 10-04-2021 - 16:06


#2
Hoang72

Hoang72

    Thiếu úy

  • Điều hành viên OLYMPIC
  • 539 Bài viết

$\boxed{1}$ Cho $a,b,c$ là các số dương thỏa

$\frac{a}{1+a}+\frac{b}{1+b}+\frac{c}{1+c}=1$
Chứng minh rằng:  $abc \leq \frac{1}{8}$

Quy đồng chắc cũng được:

Ta có $\frac{a}{1+a}+\frac{b}{1+b}+\frac{c}{1+c}=1\Leftrightarrow \frac{\sum a(1+b)(1+c)}{(a+1)(b+1)(c+1)}=1\Leftrightarrow ab+bc+ca+2abc=1$.

Giả sử $abc>\frac{1}{8}\Rightarrow ab+bc+ca\geq 3\sqrt[3]{ab.bc.ca}>\frac{3}{4}\Rightarrow ab+bc+ca+2abc>1$ (vô lí).

Vậy $abc\leq\frac{1}{8}$.



#3
pcoVietnam02

pcoVietnam02

    Thượng sĩ

  • Thành viên
  • 208 Bài viết

Bài toán tiếp theo: 

$\boxed{2}$ Chứng minh với 4 số thực không âm bất kì $a,b,c,d$ ta có:

$$(ab)^{\frac{1}{3}}+(cd)^{\frac{1}{3}} \leq [(a+c+b)(a+c+d)]^{\frac{1}{3}}$$


Bài viết đã được chỉnh sửa nội dung bởi pcoVietnam02: 07-04-2021 - 15:42


#4
ChiMiwhh

ChiMiwhh

    Trung sĩ

  • Thành viên
  • 126 Bài viết

Bảng điểm:
Hoang72: 1 điểm

Bài toán tiếp theo:
$\boxed{2}$ Chứng minh với 4 số thực không âm bất kì $a,b,c,d$ ta có:
$(ab)^{\frac{1}{3}}+(cd)^{\frac{1}{3}} \geq [(a+c+b)(a+c+d)]^{\frac{1}{3}}$


Nếu như em ko nhầm thì nó sai với $a=b=c=d=1$
P.s: em dùng dth nên ko chỉnh được, lát e chỉnh lại sau ạ

#5
ChiMiwhh

ChiMiwhh

    Trung sĩ

  • Thành viên
  • 126 Bài viết

Bài toán tiếp theo: 

$\boxed{2}$ Chứng minh với 4 số thực không âm bất kì $a,b,c,d$ ta có:

$(ab)^{\frac{1}{3}}+(cd)^{\frac{1}{3}} \leq [(a+c+b)(a+c+d)]^{\frac{1}{3}}$

Bài viết trích dẫn bđt trong sol https://diendantoanh...r-và-minkowski/

Thật may mắn khi phone của e có thể chuyển qua desktop mode

P.s: sr mng vì em áp dụng bđt phụ sai

IMG_20210406_163653_edit.jpg


Bài viết đã được chỉnh sửa nội dung bởi ChiMiwhh: 06-04-2021 - 18:12


#6
Hoang72

Hoang72

    Thiếu úy

  • Điều hành viên OLYMPIC
  • 539 Bài viết

Thật may mắn khi phone của e có thể chuyển qua desktop mode

attachicon.gifIMG_20210406_163653_edit.jpg

Chắc có gì đó nhầm lần vì điểm rơi là $b=d=2a=2c$.



#7
ChiMiwhh

ChiMiwhh

    Trung sĩ

  • Thành viên
  • 126 Bài viết

Chắc có gì đó nhầm lần vì điểm rơi là $b=d=2a=2c$.

Vâng ạ


Bài viết đã được chỉnh sửa nội dung bởi ChiMiwhh: 06-04-2021 - 17:40


#8
pcoVietnam02

pcoVietnam02

    Thượng sĩ

  • Thành viên
  • 208 Bài viết

Gợi ý bài số 2 (làm theo cách này được 0,5đ) : Phân tích  $\frac{ab}{(a+c+b)(a+c+d)}$ để sử dụng bất đẳng thức $AM-GM$ cho bộ $3$ số không âm. Tương tự với $\frac{cd}{(a+c+b)(a+c+d)}$ rồi cộng các bất đẳng thức lại.


Bài viết đã được chỉnh sửa nội dung bởi pcoVietnam02: 07-04-2021 - 16:35


#9
yungazier

yungazier

    Lính mới

  • Thành viên mới
  • 6 Bài viết

Bài toán tiếp theo: 

$\boxed{2}$ Chứng minh với 4 số thực không âm bất kì $a,b,c,d$ ta có:

$(ab)^{\frac{1}{3}}+(cd)^{\frac{1}{3}} \leq [(a+c+b)(a+c+d)]^{\frac{1}{3}}$

Áp dụng bđt phụ: $\sqrt[3]{abc}+\sqrt[3]{xyz}\leq \sqrt[3]{(a+x)(b+y)(c+z)}$

Chứng minh BĐT: BĐT tương đương $\sqrt[3]{\frac{abc}{(a+x)(b+y)(c+z)}}+\sqrt[3]{\frac{xyz}{(a+x)(b+y)(c+z)}}\leq 1$

Theo AM-GM 3 số,ta có VT$\leq \frac{1}{3}(\frac{a}{a+x}+\frac{b}{b+y}+\frac{c}{c+z})+\frac{1}{3}(\frac{x}{a+x}+\frac{y}{b+y}+\frac{z}{c+z})=1$

Quay trở lại lại bài toán: $LHS=\sqrt[3]{ab}+\sqrt[3]{cd}=\sqrt[3]{(a+c).b.\frac{a}{a+c}}+\sqrt[3]{d.(a+c).\frac{c}{a+c}}\leq \sqrt[3]{(a+c+d).(a+c+b).\frac{a+c}{a+c}}=RHS$



#10
pcoVietnam02

pcoVietnam02

    Thượng sĩ

  • Thành viên
  • 208 Bài viết

Áp dụng bđt phụ: $\sqrt[3]{abc}+\sqrt[3]{xyz}\leq \sqrt[3]{(a+x)(b+y)(c+z)}$

Chứng minh BĐT: BĐT tương đương $\sqrt[3]{\frac{abc}{(a+x)(b+y)(c+z)}}+\sqrt[3]{\frac{xyz}{(a+x)(b+y)(c+z)}}\leq 1$

Theo AM-GM 3 số,ta có VT$\leq \frac{1}{3}(\frac{a}{a+x}+\frac{b}{b+y}+\frac{c}{c+z})+\frac{1}{3}(\frac{x}{a+x}+\frac{y}{b+y}+\frac{z}{c+z})=1$

Quay trở lại lại bài toán: $LHS=\sqrt[3]{ab}+\sqrt[3]{cd}=\sqrt[3]{(a+c).b.\frac{a}{a+c}}+\sqrt[3]{d.(a+c).\frac{c}{a+c}}\leq \sqrt[3]{(a+c+d).(a+c+b).\frac{a+c}{a+c}}=RHS$

 

Chính xác rồi. Ta sẽ tiếp tục với bài toán số 3: 

 

$\boxed{3}$ Cho $a,b,c \geq 0$ sao cho $(a+c-3)b +1 =0$. Tìm giá trị nhỏ nhất của:

$$ P=\frac{1}{a+1} + \frac{b}{a+b} + \frac{b}{ac+3b}$$



#11
KietLW9

KietLW9

    Đại úy

  • Điều hành viên THCS
  • 1737 Bài viết

Chính xác rồi. Ta sẽ tiếp tục với bài toán số 3: 

 

$\boxed{3}$ Cho $a,b,c \geq 0$ sao cho $(a+c-3)b +1 =0$. Tìm giá trị nhỏ nhất của:

$$ P=\frac{1}{a+1} + \frac{b}{a+b} + \frac{b}{ac+3b}$$

Từ giả thiết suy ra $a+\frac{1}{b}+c=3$

Đặt $(a,\frac{1}{b},c)\rightarrow (x,y,z)$ thì x + y + z = 3 và ta cần tìm GTNN của: $P=\frac{1}{x+1}+\frac{1}{xy+1}+\frac{1}{xyz+3}$

Áp dụng Cauchy-Schwarz dạng phân thức, ta được: $\frac{1}{x+1}+\frac{1}{xy+1}+\frac{1}{xyz+3}\geqslant \frac{9}{x+xy+xyz+5}$

Ta có: $x+xy+xyz=x+xy(z+1)\leqslant x +x.\frac{(y+z+1)^2}{4}=x+\frac{x(4-x)^2}{4}$

Xét: $x+\frac{x(4-x)^2}{4}-4=\frac{(x-4)(x-2)^2}{4}\leqslant 0\Rightarrow  x+\frac{x(4-x)^2}{4}\leqslant 4$

Do đó $P\geqslant \frac{9}{x+xy+xyz+5}\geqslant \frac{9}{4+5}=1$  

Đẳng thức xảy ra khi a = 2; b = 1; c = 0


Trong cuộc sống không có gì là đẳng thức , tất cả đều là bất đẳng thức  :ukliam2:   :ukliam2: 

 

 

$\text{LOVE}(\text{KT}) S_a (b - c)^2 + S_b (c - a)^2 + S_c (a - b)^2 \geqslant 0\forall S_a,S_b,S_c\geqslant 0$

 

 

 


#12
pcoVietnam02

pcoVietnam02

    Thượng sĩ

  • Thành viên
  • 208 Bài viết

Từ giả thiết suy ra $a+\frac{1}{b}+c=3$

Đặt $(a,\frac{1}{b},c)\rightarrow (x,y,z)$ thì x + y + z = 3 và ta cần tìm GTNN của: $P=\frac{1}{x+1}+\frac{1}{xy+1}+\frac{1}{xyz+3}$

Áp dụng Cauchy-Schwarz dạng phân thức, ta được: $\frac{1}{x+1}+\frac{1}{xy+1}+\frac{1}{xyz+3}\geqslant \frac{9}{x+xy+xyz+5}$

Ta có: $x+xy+xyz=x+xy(z+1)\leqslant x +x.\frac{(y+z+1)^2}{4}=x+\frac{x(4-x)^2}{4}$

Xét: $x+\frac{x(4-x)^2}{4}-4=\frac{(x-4)(x-2)^2}{4}\leqslant 0\Rightarrow  x+\frac{x(4-x)^2}{4}\leqslant 4$

Do đó $P\geqslant \frac{9}{x+xy+xyz+5}\geqslant \frac{9}{4+5}=1$  

Đẳng thức xảy ra khi a = 2; b = 1; c = 0

 

Rất chính xác. Bây giờ sẽ là bài số 4:

 

$\boxed{4}$ Cho $x,y,z > \frac{1}{2}$. Chứng minh rằng:

$$      \frac{1}{2a-1}+\frac{1}{2b-1}+\frac{1}{2c-1}+\frac{4ab}{1+ab}+\frac{4bc}{1+bc}+\frac{4ac}{1+ac} \geq 9$$


Bài viết đã được chỉnh sửa nội dung bởi pcoVietnam02: 07-04-2021 - 21:42


#13
KietLW9

KietLW9

    Đại úy

  • Điều hành viên THCS
  • 1737 Bài viết

Rất chính xác. Bây giờ sẽ là bài số 4:

 

$\boxed{4}$ Cho $x,y,z \geq \frac{1}{2}$. Chứng minh rằng:

$$      \frac{1}{2a-1}+\frac{1}{2b-1}+\frac{1}{2c-1}+\frac{4ab}{1+ab}+\frac{4bc}{1+bc}+\frac{4ac}{1+ac} \geq 9$$

Bài này có sai đề không? Giả thiết đáng lẽ là $a,b,c\geqslant 1$


Trong cuộc sống không có gì là đẳng thức , tất cả đều là bất đẳng thức  :ukliam2:   :ukliam2: 

 

 

$\text{LOVE}(\text{KT}) S_a (b - c)^2 + S_b (c - a)^2 + S_c (a - b)^2 \geqslant 0\forall S_a,S_b,S_c\geqslant 0$

 

 

 


#14
pcoVietnam02

pcoVietnam02

    Thượng sĩ

  • Thành viên
  • 208 Bài viết

Bài này có sai đề không? Giả thiết đáng lẽ là $a,b,c\geqslant 1$

 

À giả thiết là $x,y,z > \frac{1}{2}$ nha để anh sửa lại.



#15
Hoang72

Hoang72

    Thiếu úy

  • Điều hành viên OLYMPIC
  • 539 Bài viết

Rất chính xác. Bây giờ sẽ là bài số 4:

 

$\boxed{4}$ Cho $x,y,z > \frac{1}{2}$. Chứng minh rằng:

$$      \frac{1}{2a-1}+\frac{1}{2b-1}+\frac{1}{2c-1}+\frac{4ab}{1+ab}+\frac{4bc}{1+bc}+\frac{4ac}{1+ac} \geq 9$$

Đây là bài T7/524. Nhác đánh quá nên em gửi file bài làm của em gửi tạp chí luôn:

File gửi kèm

  • File gửi kèm  T7_524.pdf   273.45K   421 Số lần tải


#16
pcoVietnam02

pcoVietnam02

    Thượng sĩ

  • Thành viên
  • 208 Bài viết

Bài làm rất tốt. Bây giờ ta sẽ bắt đầu thử sức với một bài khó để thử trình độ các bạn nhé:

 

$\boxed{5}$ Cho $a,b,c$ là số dương thỏa $x^2+y^2+z^2=1$. Chứng minh rằng: 

$$ \frac{x}{1-x^2} + \frac{y}{1-y^2} +  \frac{z}{1-z^2} \geq \frac{3\sqrt{3}}{2}$$ 

 

*Chú ý: Sử dụng bất đẳng thức Jensen được 2 điểm, các phương pháp khác 1 điểm



#17
ChiMiwhh

ChiMiwhh

    Trung sĩ

  • Thành viên
  • 126 Bài viết

Bài làm rất tốt. Bây giờ ta sẽ bắt đầu thử sức với một bài khó để thử trình độ các bạn nhé:

 

$\boxed{5}$ Cho $a,b,c$ là số dương thỏa $x^2+y^2+z^2=1$. Chứng minh rằng: 

$$ \frac{x}{1-x^2} + \frac{y}{1-y^2} +  \frac{z}{1-z^2} \geq \frac{3\sqrt{3}}{2}$$ 

 

*Chú ý: Sử dụng bất đẳng thức Jensen được 2 điểm, các phương pháp khác 1 điểm

Đặt $a=\sqrt{3}x$ và tương tự nên ta có $a^2+b^2+c^2=3$

Cần cm $\sum \frac{a}{3-a^2}\geq \frac{3}{2}$

Xét bđt phụ

$\frac{a}{3-a^2}\geq \frac{a^2}{2}\Leftrightarrow (a-1)^2(a+2)\geq 0$ luôn đúng

Thiết lập tương tự và cộng lại

Xảy ra khi $x=y=z=\frac{1}{\sqrt{3}}$



#18
pcoVietnam02

pcoVietnam02

    Thượng sĩ

  • Thành viên
  • 208 Bài viết

Đặt $a=\sqrt{3}x$ và tương tự nên ta có $a^2+b^2+c^2=3$

Cần cm $\sum \frac{a}{3-a^2}\geq \frac{3}{2}$

Xét bđt phụ

$\frac{a}{3-a^2}\geq \frac{a^2}{2}\Leftrightarrow (a-1)^2(a+2)\geq 0$ luôn đúng

Thiết lập tương tự và cộng lại

Xảy ra khi $x=y=z=\frac{1}{\sqrt{3}}$

 

Quá nhanh :)) Nhưng bây giờ bài này vẫn ngoại lệ là ai giải bằng cách sử dụng bđt Jensen vẫn cho 1 điểm. Nếu không giải được thì sẽ có đáp án vào ngày mai. Bây giờ là bài tiếp theo: 

 

$\boxed{6}$ Cho các số thực $a,b,c$ thỏa mãn $a^2+b^2+c^2=1$. Chứng minh rằng:

$$(a-b)(b-c)(c-a)(a+b+c) \geq \frac{-9\sqrt{2}}{32}$$

 

@Update: Lời giải của bài này sẽ được update sau 21h kèm câu số 7.


Bài viết đã được chỉnh sửa nội dung bởi pcoVietnam02: 09-04-2021 - 15:40


#19
KietLW9

KietLW9

    Đại úy

  • Điều hành viên THCS
  • 1737 Bài viết

Quá nhanh :)) Nhưng bây giờ bài này vẫn ngoại lệ là ai giải bằng cách sử dụng bđt Jensen vẫn cho 1 điểm. Nếu không giải được thì sẽ có đáp án vào ngày mai. Bây giờ là bài tiếp theo: 

 

$\boxed{6}$ Cho các số thực $a,b,c$ thỏa mãn $a^2+b^2+c^2=1$. Chứng minh rằng:

$$(a-b)(b-c)(c-a)(a+b+c) \geq \frac{-9\sqrt{2}}{32}$$

 

@Update: Lời giải của bài này sẽ được update sau 21h kèm câu số 7.

Áp dụng AM - GM, ta được: $[3(a^2+b^2+c^2)]^2=[2(a-b)^2+2(a-c)(b-c)+(a+b+c)^2]^2\geqslant 8|(a-c)(b-c)|[2(a-b)^2+(a+b+c)^2]\geqslant 16\sqrt{2}|(a-b)(b-c)(c-a)(a+b+c)|\Rightarrow (a-b)(b-c)(c-a)(a+b+c)\geqslant \frac{-9\sqrt{2}}{32}(Q.E.D)$     

Chú ý rằng:  Từ đây ta cũng có GTLN của $(a-b)(b-c)(c-a)(a+b+c)$ là $\frac{9\sqrt{2}}{32}$


Bài viết đã được chỉnh sửa nội dung bởi KietLW9: 09-04-2021 - 21:58

Trong cuộc sống không có gì là đẳng thức , tất cả đều là bất đẳng thức  :ukliam2:   :ukliam2: 

 

 

$\text{LOVE}(\text{KT}) S_a (b - c)^2 + S_b (c - a)^2 + S_c (a - b)^2 \geqslant 0\forall S_a,S_b,S_c\geqslant 0$

 

 

 


#20
pcoVietnam02

pcoVietnam02

    Thượng sĩ

  • Thành viên
  • 208 Bài viết

Áp dụng AM - GM, ta được: $[3(a^2+b^2+c^2)]^2=[2(a-b)^2+2(a-c)(b-c)+(a+b+c)^2]^2\geqslant 8(a-c)(b-c)[2(a-b)^2+(a+b+c)^2]\geqslant 16\sqrt{2}|(a-c)(b-c)(c-a)(a+b+c)|\Rightarrow (a-b)(b-c)(c-a)(a+b+c)\geqslant \frac{-9\sqrt{2}}{32}(Q.E.D)$     

 

Bài làm đi đúng hướng nên tất nhiên đáp án cũng chính xác. Bây giờ là bài số 7, bây giờ ta sẽ khởi động nhẹ với những bài bất đẳng thức lượng giác. Nếu sử dụng Jensen và chứng minh đây đủ được 2 điểm, những cách làm đơn thuần được 1 điểm.

 

$\boxed{7}$ Chứng minh với mọi tam giác ABC, ta có:

$a)$ $sin$ $A$ + $sin$ $B$ + $sin$ $C$ $\leq \frac{3\sqrt{3}}{2}$

$b)$ $tan$ $A$ + $tan$ $B$ + $tan$ $C$ $\geq$ $3\sqrt{3}$   (góc$A,B,C$ nhọn)


Bài viết đã được chỉnh sửa nội dung bởi pcoVietnam02: 09-04-2021 - 21:59






Được gắn nhãn với một hoặc nhiều trong số những từ khóa sau: bất đẳng thức, marathon, cấp 2, cấp 3, imo, vmo, olympic, tuyển sinh 10

1 người đang xem chủ đề

0 thành viên, 1 khách, 0 thành viên ẩn danh